• We need your support!

    We are currently struggling to cover the operational costs of Xtremepapers, as a result we might have to shut this website down. Please donate if we have helped you and help make a difference in other students' lives!
    Click here to Donate Now (View Announcement)

Maths, Addmaths and Statistics: Post your doubts here!

Messages
1,388
Reaction score
2,536
Points
273
Firstlly you shoud have drawn the graph and all.....
now if u let y=1+2x^2 -x^3
therefore y=k as well
now the line y=k is a obviously a horizontal line......
so what you do is find the range from which this line y=k will meet the curve 3 times.....
you will see from the graph of prev part that between the point y=1 and y=2 any horizontal line between these points will meet the curve thrice
SOOOO
1<k<2
k lies between 1 and 2
 
Messages
108
Reaction score
19
Points
28
Firstlly you shoud have drawn the graph and all.....
now if u let y=1+2x^2 -x^3
therefore y=k as well
now the line y=k is a obviously a horizontal line......
so what you do is find the range from which this line y=k will meet the curve 3 times.....
you will see from the graph of prev part that between the point y=1 and y=2 any horizontal line between these points will meet the curve thrice
SOOOO
1<k<2
k lies between 1 and 2

Thank you so much! :)
 
Messages
3,225
Reaction score
15,036
Points
523
Firstlly you shoud have drawn the graph and all.....
now if u let y=1+2x^2 -x^3
therefore y=k as well
now the line y=k is a obviously a horizontal line......
so what you do is find the range from which this line y=k will meet the curve 3 times.....
you will see from the graph of prev part that between the point y=1 and y=2 any horizontal line between these points will meet the curve thrice
SOOOO
1<k<2
k lies between 1 and 2

>_< Damn! I left that part in my exam because I couldn't recall this y=k and intersection etc

Btw Thankyou :) Now I wont repeat this mistake in CIE. IA
 
Messages
343
Reaction score
220
Points
53
Firstlly you shoud have drawn the graph and all.....
now if u let y=1+2x^2 -x^3
therefore y=k as well
now the line y=k is a obviously a horizontal line......
so what you do is find the range from which this line y=k will meet the curve 3 times.....
you will see from the graph of prev part that between the point y=1 and y=2 any horizontal line between these points will meet the curve thrice
SOOOO
1<k<2
k lies between 1 and 2
is this maths or add maths?
 
Messages
492
Reaction score
399
Points
73
Please anyone explain me (Q7-ii-b). I know the coordinates but am confused about the nature of the stationary point with a modulus function. How on earth can it be maximum?
 

Attachments

  • Nov 06.pdf
    159.6 KB · Views: 3
Messages
1,388
Reaction score
2,536
Points
273
Please anyone explain me (Q7-ii-b). I know the coordinates but am confused about the nature of the stationary point with a modulus function. How one earth can it be maximum?
well modulus function means reflecting the graph of f(x) in the x-axis......
now automatically the minimum point will become a maximum
(since a reflection of a minimum pt in horizontal line gives a maximum point)
 
Messages
492
Reaction score
399
Points
73
well modulus function means reflecting the graph of f(x) in the x-axis......
now automatically the minimum point will become a maximum
(since a reflection of a minimum pt in horizontal line gives a maximum point)
But there are coordinates of y which are greater than 11.....:(
 
Top